Craniofacial Flashcards

(74 cards)

1
Q

A 2-month-old female infant is brought in by her parents for evaluation of a 4-cm,
enlarging infantile hemangioma on her left cheek that does not obstruct her vision.
Which of the following is the most appropriate management?

A ) Intralesional corticosteroid injection
B ) Laser ablation
C ) Oral propanolol therapy
D ) Surgical excision
E ) Observation

A

The correct response is Option C.

Systemic propanolol therapy has quickly become a safe and effective treatment for
hemangioma and is well tolerated with few side effects. It is considered the first line
medical treatment of infantile hemangiomas at many medical centers. Surgical treatment is
generally reserved for lesions that are refractory to medical management which present a
functional impairment such as affecting or obstructing vision. Laser ablation and
intralesional corticosteroid injection are less effective in several meta-analyses and some
randomized controlled trials. Observation is reasonable for small, stable lesions in
cosmetically non-sensitive areas.

2019

How well did you know this?
1
Not at all
2
3
4
5
Perfectly
2
Q

Cranial bone graft - when does diploic space begin to form in kid?

A

Approximately 5 years old

How well did you know this?
1
Not at all
2
3
4
5
Perfectly
3
Q

An 8-year-old boy is brought to the office with a congenital abnormality of the ear. A
photograph is shown. When the ear is pulled on traction, the upper pole cartilage
becomes visible under the skin. This abnormality is most likely caused by which of the following?
-pic looks like cryptotia-

A ) Anomaly of the intrinsic postauricular muscles
B ) Effacement of the scaphofossa
C ) Formation of a third antihelical crus
D ) Hemifacial microsomia
E ) Intrauterine pressure

A

Correct answer is A.

This patient has cryptotia of the ear. It is also known as pocket ear. The upper part of the ear
is adherent and the cartilage is buried under the skin in a pocket. Other deformities may be
present, such as a missing upper sulcus, underdeveloped scapha, and antihelical crura.
It is caused by an anomaly of the intrinsic oblique and transverse auricular muscles.
Surgical treatment requires release from the pocket and resurfacing of the post- and
retroauricular defects. In some cases otoplasty may be required for normalization. A number
of techniques are described.

Intrauterine pressure may cause ear deformities, which may spontaneously resolve or are amenable to neonatal molding techniques. It does not cause cryptotia.
Stahl ear is associated with formation of a third antihelical crus.

Hemifacial microsomia is associated with microtia. Microtia is sometimes the only
manifestation of hemifacial microsomia, but subtle clinical findings are often present, such
as mild facial nerve weakness or soft tissue hypoplasia on the involved side.

How well did you know this?
1
Not at all
2
3
4
5
Perfectly
4
Q

A 47-year-old Caucasian man comes to the office regarding a painful enlarging mass at
the base of the tongue. He does not smoke cigarettes. The lesion measures 4.5 cm. A
biopsy of the lesion is performed and shows (+) p16 staining, nonkeratinized squamous
cell carcinoma. Further imaging and workup demonstrate an ipsilateral solitary lymph
node measuring 2.3 cm. No distal metastatic disease is found. Which of the following
best describes the stage of his disease?

A ) Stage 1
B ) Stage 2
C ) Stage 3
D )Stage 4

A

The correct response is Option B.
The correct answer is Stage 2. Previously, this patient would have been Stage 3. The
American Joint Committee on Cancer (AJCC) revised its staging system for squamous cell
cancers that stain p16 positive. These lesions are related to the human papillomavirus (HPV) and have been found to be less virulent tongue base or oropharyngeal cancers. Recent studies have demonstrated that 5 year survival difference for patients with
Stage 4 disease as <50% for HPV-negative patients and >70% for HPV-positive patients,
thus prompting the AJCC to study and revise the staging system for HPV-positive
oropharyngeal cancers. These lesions tend to be more sensitive to radiation therapy and
chemoradiation and a better prognosis overall. Patients with HPV-related squamous cell
cancers tend to be younger, male, and Caucasian. HPV-related squamous cell cancers now represent the majority of newly diagnosed oropharyngeal carcinomas in the United States.
This new staging system for HPV (+) related cancers went into effect 1/1/2017.

How well did you know this?
1
Not at all
2
3
4
5
Perfectly
5
Q

A 22-year-old man is evaluated because of a painless, firm, unilateral enlarging mass
of the body of the mandible. He denies trauma to the area and he has excellent oral
hygiene. His dentist performed fine needle biopsy that showed multinucleated giant
cells. CT scan shows a radiolucent bone lesion with an expanded cortex. Which of the
following is the most appropriate next step in management?

A ) Incisional biopsy
B ) Partial mandibulectomy with free margins
C ) Radiation therapy
D ) Resection and curettage
E ) Sclerotherapy

A

The correct response is Option D.
This patient has an aneurysmal bone cyst (ABC). These lesions may be related to giant cell
granulomas. These lesions are most common in the long bones with 1.9% of them being
reported in the mandible.
The correct answer is resection and curettage. In a study of 120 ABC cases, resection and
curettage was reported to have a 91.8% success rate (recurrence occurred in 11 out of 120
cases). Incomplete resection is hypothesized to be a cause of recurrence. Recurrence was
not related to histopathologic parameters.
Pathologically, these lesions are a pseudocyst comprised of multinucleated giant cells,
woven trabecular bone with caverns, and sinusoids lacking endothelium. Recurrence can be treated with repeat excision curettage, open packing, or block resection. These lesions are
quite vascular and typically bleed until resected, so expeditious removal is recommended
(transfusion with packed red blood cells has been reported in the literature).
Incisional biopsy would be diagnostic but is not recommended for these lesions given their
vascularity and the surgeon’s inability to control the bleeding.
Sclerotherapy is recommended for vascular malformations such as arteriovenous
malformations, venous malformations, or lymphatic malformations. This lesion would not
be responsive to this type of therapy. Partial mandibulectomy with free margins is the preferred treatment for lesions like
ameloblastomas. Given the efficacy of excision and curettage, mandibulectomy is not the
recommended first line treatment for this diagnosis. It can be employed in recurrences
(although as stated above, less aggressive interventions are typically employed first).

How well did you know this?
1
Not at all
2
3
4
5
Perfectly
6
Q

In a patient undergoing reconstructive cranioplasty, an increased rate of complications
is most likely if which of the following is present?

A ) Frontal location
B ) Occipital location
C ) Parietal location
D ) Sphenoidal location
E ) Temporal location

A

The correct response is Option A.

Early decompressive craniectomy is a life-saving maneuver for certain traumatic
brain injuries and can be performed far forward in the theater of war. Patients
treated with decompressive craniectomy for combat injuries are a unique
understudied population. Outcome of treatment of this patient cohort has been
previously reported using a standardized cranial defect treatment protocol using
custom alloplast implants. Two subgroups of patients (large endocranial dead
space and frontal orbital bar injuries) were identified as often having higher rates of
complications than other cranial reconstruction cohorts.

How well did you know this?
1
Not at all
2
3
4
5
Perfectly
7
Q

Which of the following structures contributes to the formation of the tragus?

A ) First branchial arch
B ) First branchial cleft
C ) Second branchial arch
D ) Second branchial cleft

A

The correct response is Option A.

The first branchial arch contributes to the formation of the tragus and anterior helix.
The first branchial cleft is incorrect. It gives rise to the external auditory canal.
The second branchial arch is incorrect. It contributes to the formation of the majority of the
external ear – the antitragus, remainder of the helix, antihelix, and crura all arise from the
second branchial arch.
The second branchial cleft is incorrect. It is typically obliterated during development, but
may persist in the form of a second branchial cleft cyst.

How well did you know this?
1
Not at all
2
3
4
5
Perfectly
8
Q

An 18-month-old child is brought to the office after undergoing fronto-orbital
advancement for metopic craniosynostosis. Which of the following is the earliest age
the surgeon can order the x-ray studies and expect to be able to see frontal sinus
development?

A ) 1 year
B ) 2 years
C ) 4 years
D ) 6 years
E ) 10 years

A

The correct response is Option D.

Frontal sinus development is associated with specific age-related periods of growth of the
skull. The frontal sinus is absent at birth and during the initial phase of growth of the skull.
The sinus is visible only in x-ray studies at the end of the first period of skull growth. This
is the time when the endocranial table of the skull ceases to grow and conforms to the
general shape of the brain. This is not seen on x-ray studies until 6 years of age or 72
months.

How well did you know this?
1
Not at all
2
3
4
5
Perfectly
9
Q

A female infant is born with severe Treacher Collins syndrome and bilateral Pruzansky
III mandible (absence of condyle). Tracheostomy is performed for respiratory distress.

Which of the following surgeries is most likely to allow decannulation?
A ) Alloplastic condylar reconstruction
B ) Bilateral sagittal split osteotomy
C ) Costochondral rib grafts
D ) Mandibular distraction
E ) Tongue-lip adhesion

A

The correct response is Option C.

Patients with Treacher Collins syndrome may have a varied presentation. The mandible
hypoplasia may be mild or severe. In this case the patient has no temporomandibular fossa
or condyles. Tongue-lip adhesion and mandibular distraction are used in severe Pierre
Robin sequence. Because of the absence of condyles and temporal mandibular joints,
distraction, and sagittal split osteotomy are not the best options. A costochondral graft will
provide better airway support and can also be distracted in the future.

How well did you know this?
1
Not at all
2
3
4
5
Perfectly
10
Q

Which of the following statements is correct about Tessier clefts No. 3, No. 4, and No. 5?

A ) Tessier No. 3 involves the alveolar ridge, while Tessier No. 5 does not
B ) Tessier No. 3 is medial to the infraorbital nerve, while Tessier No. 4 is lateral
C ) Tessier No. 3 only affects the oral region, while Tessier No. 4 only affects the orbital
region
D ) Tessier No. 4 involves the piriform aperture, while Tessier No. 5 does not
E ) Tessier No. 4 is medial to the infraorbital nerve, while Tessier No. 5 is lateral

A

The correct response is Option E.

Tessier No. 3 and No. 4 are medial to the infraorbital nerve, but Tessier No. 5 is lateral.
Tessier No. 3 involves clefts of the nose, orbit, and lip (naso-oral-ocular cleft), whereas
Tessier No. 4 involves the lip and orbit (oral-ocular cleft), and the nose is uninvolved.
Tessier No. 5 involves oral, cheek (maxillary sinus), and orbital cleft and is the rarest.

How well did you know this?
1
Not at all
2
3
4
5
Perfectly
11
Q

A newborn male is brought to the tertiary multidisciplinary referral center for evaluation
of anorectal malformation, tracheoesophageal fistula and absent right thumb. Which of
the following associated VACTERL diagnoses is the best predictor of inpatient
mortality?

A ) Aniridia with brain stem hypoplasia
B ) Anomalies of spine or vertebrae
C ) Cardiac disease
D ) Renal or urinary anomaly
E ) Tracheal stenosis with strido

A

The correct response is Option C.

Anomalies of the spine or vertebrae (V), anorectal malformations (A), congenital cardiac
anomalies (C), esophageal atresia/tracheoesophageal fistula (TE), renal and urinary abnormalities (R), and limb lesions (L) frequently co-occur and are recognized as
VACTERL anomalies. VACTERL association is typically diagnosed in the presence of at
least three characteristic features in the absence of evidence for an overlapping condition, and is estimated to occur in approximately 1 in every 10,000 to 40,000 live births. The presence of either anorectal malformation or esophageal atresia alone generally triggers a
workup for associated VACTERL diagnoses because of their significant impact on
morbidity and mortality. For example, in a large cohort of children undergoing surgical
repair of anorectal malformations, Lautz et al. found associated VACTERL diagnoses
including congenital heart disease in 40.4%, renal or internal urinary disease in 34.7%,
spinal or vertebral anomalies in 31.4%, esophageal atresia/tracheoesophageal fistula in 7%, and limb defects in 5.6%. The most common limb defects in VACTERL association include
poorly developed or missing thumbs, or underdeveloped forearms and hands, polydactyly,
syndactyly, and reduction deformities of the lower limb.

Independent predictors of mortality in any patient with VACTERL association include
congenital heart disease (greatest for those who require cardiac surgery than those with a
diagnosis but no operation), birthweight < 2 kg, and black race. Of note, the association
between cardiac disease and higher mortality has been reproduced in several studies.
Aniridia, brain stem hypoplasia, and tracheal stenosis with stridor are not primary
characteristics of VACTERL association.

2019

How well did you know this?
1
Not at all
2
3
4
5
Perfectly
12
Q

Patients with unilateral cleft lip and associated nasal deformities have each of the following findings EXCEPT

(A) attenuation and inferior positioning of the lower lateral cartilage on the side of the cleft
(B) elongation of the philtrum
(C) insertion of the orbicularis oris muscle into the cleft margin and alar wing
(D) outward rotation and projection of the premaxilla
(E) unilateral shortening of the columella

A

The correct response is Option B.

In patients who have unilateral cleft lip and associated nasal deformities, the premaxilla is rotated and projected outward, and the lateral maxillary element is collapsed and retropositioned. The inferior edge of the septum lies outside of the vomer groove, while the nasal spine is located in the floor of the normal nostril. The affected columella is 25% to 50% shorter than the unaffected side. The lower lateral cartilage is attenuated, and the nasal dome lies separate, below the opposite cartilage. The alar base is flared and rotated outward, and the vestibular lining is deficient on the side of the cleft.

With regard to the lip deformities, the philtrum termination of the orbicularis oris muscle in the lateral lip is shortened at the margin of the cleft; at this point, the muscle inserts into the alar wing. The muscles between the philtral midline and the cleft are hypoplastic. Two thirds of the cupid’s bow is preserved, as well as one philtral column and a dimple hollow.

How well did you know this?
1
Not at all
2
3
4
5
Perfectly
13
Q

A 15-year-old patient with a left unilateral cleft lip and palate is evaluated because of a congenitally absent left lateral incisor. The patient’s orthodontist recommended orthodontic canine substitution in the lateral incisor space rather than space maintenance. Relative to patients who do not undergo canine substitution, which of the following best describes this patient’s post-orthodontic risk for malocclusion?

A) Decreased risk for Angle class II malocclusion
C) Decreased risk for Angle class III malocclusion
C) Increased risk for Angle class II malocclusion
D) Increased risk for Angle class III malocclusion
E) No change in risk for Angle class malocclusion

A

The correct response is Option D.

Patients with complete cleft lip that includes the alveolus have a higher risk for lateral incisor tooth agenesis. Both congenital absence and orthodontic canine substitution are independently associated with an increased risk for maxillary hypoplasia, Angle class III malocclusion, and the subsequent need for orthognathic surgery.

2024

How well did you know this?
1
Not at all
2
3
4
5
Perfectly
14
Q

A 17-year-old girl with a history of bilateral cleft lip and palate comes for evaluation. Physical examination shows severe mid face deficiency and Angle class III malocclusion with 10 mm of negative overjet. Le Fort I distraction using an internal distractor is recommended. Compared with conventional Le Fort I maxillary advancement, Le Fort I maxillary distraction is most likely to result in which of the following for this patient?

A) Decreased complication rate
B) Decreased risk for relapse
C) Improved psychosocial outcomes
D) Increased predictability of final occlusion
E) Increased risk for postoperative speech dysfunction

A

The correct response is Option B.

Le Fort I distraction is a powerful tool to address Angle class III malocclusion in patients with cleft lip and palate. Numerous studies have shown that distraction yields improved speech outcomes when controlling for the degree of advancement, while also decreasing the rate of relapse, as compared with conventional distraction. However, final occlusion with internal maxillary distraction is less predictable than external distraction or conventional maxillary advancement surgery. Complication rates and psychosocial outcomes have not been shown to differ between these techniques.

2024

How well did you know this?
1
Not at all
2
3
4
5
Perfectly
15
Q

An otherwise healthy 8-year-old girl is referred by her pediatrician because of poor speech intelligibility that is affecting her school performance. Physical examination shows no abnormalities in the patient’s facial features. Intraoral examination shows a bluish discoloration in the midline palate. On speech analysis, the patient is hypernasal and unable to produce the consonants p, b, and g. Which of the following is the most likely diagnosis?

A) Arterial venous malformation
B) Chronic tonsillitis
C) Palatal neoplasm
D) Submucous cleft palate
E) Torus palatini

A

The correct response is Option D.

Patients with submucous cleft palate may have delayed presentation to cleft centers due to undiagnosed disease. They might be considered developmentally behind and have behavioral issues given their inability to express themselves and be understood. It is not uncommon to miss their abnormal soft palate anatomy, which can be subtly V-shaped and have the midline bluish line (pellucid zone).

Patients with arterial venous malformation of this area do not necessarily produce hypernasal speech. Chronic tonsillitis can be associated with hyponasality, enlarged tonsils, and bad breath. Palatal neoplasm is usually slow-growing and not associated with a pellucid zone or long-term speech issues. Torus palatini does not produce hypernasal speech.

2024

How well did you know this?
1
Not at all
2
3
4
5
Perfectly
16
Q

In a patient with a cleft lip and palate, the congenital absence of which of the following permanent maxillary teeth is most common?

A) Canine
B) Central incisor
C) First premolar
D) Lateral incisor
E) Second premolar

A

The correct response is Option D.

Dental anomalies are frequent in patients with cleft lip and palate. The maxillary lateral incisor is the most frequently congenitally absent secondary tooth, missing in around 60% of patients.

The other teeth listed are missing at a much lower incidence.

2024

How well did you know this?
1
Not at all
2
3
4
5
Perfectly
17
Q

A 15-year-old boy with repaired left unilateral cleft lip and palate undergoes alveolar bone grafting. The patient did not undergo preoperative orthodontic preparation. He was unable to tolerate oral intake for 15 hours after surgery, and no postoperative antibiotics were administered. Which of the following is the strongest predictor of graft failure in this patient?

A) Age greater than 12 years
B) Lack of orthodontic preparation with maxillary arch expansion
C) No postoperative antibiotic regimen
D) Time to postoperative oral intake greater than 12 hour

A

The correct response is Option A.

Secondary bone grafting is widely accepted in cleft care; however, there is limited consensus on the technique, timing, and perioperative management of the cases. In general, the age of the patient at secondary bone grafting is typically at the time of mixed dentition and ranges from 8 to 12 years of age. Several studies have demonstrated that later age at time of bone grafting is associated with higher rates of graft failure, especially when the canine has already erupted.

Lack of orthodontic preparation and postoperative antibiotic regimen were not predictors of graft failure. Time to postoperative oral intake was also not predictive of graft failure.

2024

How well did you know this?
1
Not at all
2
3
4
5
Perfectly
18
Q

A 1-month-old female infant born with a cleft palate and micrognathia is brought for evaluation of consistent desaturations. The patient undergoes placement of mandibular distractors, and postoperative examination shows asymmetry of the lower lip when she cries. Which of the following is the most likely cause of this patient’s current symptoms?

A) Buccal nerve palsy
B) Hardware failure
C) Marginal mandibular nerve injury
D) Orbicularis oris muscle injury
E) Zygomatic nerve palsy

A

The correct response is Option C.

The marginal mandibular nerve is a branch of the facial nerve that innervates the depressor labii inferioris muscle, depressor anguli oris muscle, and mentalis muscle. A patient with injury to the marginal mandibular branch of the facial nerve presents with a deformity opening the mouth, smiling or grimacing. Injury to the marginal mandibular nerve causes paralysis of muscles of the lower lip of that side.

The other options presented would not produce weakness of the depressor labii inferioris muscle.

2024

How well did you know this?
1
Not at all
2
3
4
5
Perfectly
19
Q

A 10-month-old female infant is evaluated for cleft palate reconstruction. A traditional Furlow palatoplasty is planned with the flap labeled “A” in the preoperative photograph shown. Which of the following best describes the tissue of this flap and the direction it will transpose?

“A” is the left side of patient’s designed Z

Tissue type Transposition direction
A) Mucosa and muscle      anteriorly
B) Mucosa and muscle      posteriorly
C) Mucosa only         anteriorly
D) Mucosa only         posteriorly

A

The correct response is Option B.

In a traditional Furlow palatoplasty, the flap labeled “A” includes the levator veli palatini muscle as a musculomucosal flap, and it is transposed posteriorly in the final Z-plasty.

2024

How well did you know this?
1
Not at all
2
3
4
5
Perfectly
20
Q

A 2-year-old boy who was born with bilateral cleft lip and palate and recently adopted internationally is brought by his parents for evaluation. The cleft lip was repaired before adoption, but the palate was not repaired. A photograph is shown. The parents would like to discuss management options for the best functional and aesthetic outcome for their child and ease of social integration. Which of the following is the most appropriate next step in management?

A) Cleft rhinoplasty
B) Complete cleft lip revision
C) Repair of the cleft palate
D) Speech therapy
E) Video fluoroscopy

A

The correct response is Option C.

The timing of cleft palate repair affects speech outcome. This child is already older than optimal timing, so repair of the cleft palate to allow for generation of normal speech is the correct next step. There is no reason for speech therapy or video fluoroscopy since the palate is not repaired yet. Primary nasal repair is done at the time of original lip repair, and revisions depend on the degree of deformity, but the definitive rhinoplasty is deferred until skeletal maturity. A complete lip revision for this patient’s hypertrophic scar can improve the appearance of this child but should wait until after palate repair.

2024

How well did you know this?
1
Not at all
2
3
4
5
Perfectly
21
Q

A 7-year-old boy with 22q11.2 deletion syndrome (DiGeorge syndrome) and a previously repaired submucous cleft palate with persistent velopharyngeal insufficiency is scheduled to undergo posterior pharyngeal flap surgery for speech improvement. Awareness of which of the following anatomical differences in this patient will ensure the safest repair possible?

A) Abnormal course of glossopharyngeal nerve
B) Extra-anatomic tonsillar tissues
C) Inferiorly positioned maxillary artery
D) Medial positioning of internal carotid arteries
E) Superior pharyngeal constrictor hypertrophy

A

The correct response is Option D.

In patients with 22q11.2 deletion syndrome (DiGeorge syndrome) with velopharyngeal insufficiency that persists after palatoplasty, posterior pharyngeal flap surgery is an option, but it must be performed carefully because these patients tend to have medialized internal carotid arteries, and these can be injured during dissection of the posterior pharyngeal flap.

The other options are incorrect, since these are not features of the anatomy in patients who have 22q11.2 deletions.

2024

How well did you know this?
1
Not at all
2
3
4
5
Perfectly
22
Q

A 25-year-old man is brought to the emergency department 1 hour after a motorcycle collision. The patient reports poor alignment of his teeth. Physical examination shows maxillary mobility and an anterior open bite. Which of the following muscles is most likely responsible for these findings?

A) Buccinator
B) Masseter
C) Mentalis
D) Pterygoid
E) Zygomaticus major and minor

A

The correct response is Option D.

This patient has a Le Fort I fracture. Malocclusion with an anterior open bite deformity is caused by the vector of pull of the medial and lateral pterygoid muscles. These muscles pull the fractured maxilla inferiorly and posteriorly, leading to premature posterior occlusal contact during bite. The mentalis muscle elevates the base of the lower lip. The masseter muscles elevate the mandible and pull it anteriorly. The zygomaticus muscles are muscles of facial expression and elevate the upper lip. The buccinator muscles are also muscles of facial expression and assist with chewing.

2024

How well did you know this?
1
Not at all
2
3
4
5
Perfectly
23
Q

Which of the following bones does NOT form part of the orbit?

A) Maxilla
B) Nasal
C) Palatine
D) Sphenoid
E) Zygoma

A

The correct response is Option B.

There are seven bones that make up the orbit: 1) sphenoid, 2) maxilla, 3) palatine, 4) zygoma, 5) ethmoid, 6) lacrimal, and 7) frontal. The nasal bones are not included.

How well did you know this?
1
Not at all
2
3
4
5
Perfectly
24
Q

A 13-year-old girl is brought to the clinic because of a 3-day history of headache, fever, vomiting, and swelling of the forehead and scalp. The patient reports that she sustained a head injury during a sports game 3 weeks ago. Physical examination shows no additional abnormalities. Which of the following is the most likely diagnosis?

A) Benign skin neoplasm
B) Frontal bone osteomyelitis
C) Nasal bone fracture
D) Orbital floor fracture

A

The correct response is Option B.

Frontal bone osteomyelitis with subperiosteal abscess (Pott puffy tumor) is most commonly seen in adolescents with history of head trauma, sinus infection, dental infection, and after neurosurgical procedures.

Patients who sustained orbital floor fractures may present with headache, nausea and vomiting, extraocular muscle restriction, and enophthalmos, but no forehead and scalp swelling or fever. The symptoms associated with a benign neoplasm will likely be more gradual and insidious, without fever or headache. A patient 3 weeks out from sustaining a nasal bone fracture could have deviation or headache, but would likely not have fever or swelling.

2024

How well did you know this?
1
Not at all
2
3
4
5
Perfectly
25
A 35-year-old woman is evaluated because of pain in the left cheek after sustaining an assault the previous night. A three-dimensional CT scan is shown. Which of the following additional CT findings is the strongest indicator for two-point open reduction and internal fixation in this patient? CT 3D of ZMC fx only A) Associated ipsilateral naso-orbito-ethmoid fracture B) Associated large ipsilateral orbital floor defect C) Large fragment simple fracture D) Maxillary buttress and rim comminution
The correct response is Option C. This fracture is favorable for considering two-point fixation at the zygomatic frontal and zygomatic maxillary buttresses. There is no comminution seen, and the reference points will be true. There is no associated naso-orbito-ethmoid fracture. This fracture could be addressed with two-point fixation with limited exposures of the upper lid/brow and gingivobuccal complex without needing lid incision as well. Indications for three-point fixation may include more complex and comminuted fractures with poor reference points, such as associated naso-orbito-ethmoid fracture, significant comminution, and large medial wall and floor defects with hypoglobus and enophthalmos. 2024
26
A 45-year-old man presents to the emergency department because of blurred vision and a “black eye” sustained during an assault 3 hours ago. The patient undergoes trauma workup, and extraocular movement examination shows he is unable to look upward. Maxillofacial CT scan shows an orbital fracture. Which of the following muscles is most likely to be entrapped in this patient? A) Inferior rectus B) Lateral rectus D) Medial rectus E) Superior oblique
The correct response is Option A. The inferior rectus muscle depresses and adducts the eye. Entrapment of the inferior rectus muscle causes restriction to the upward gaze. Blow-out fracture is most commonly caused by blunt trauma. Orbital blow-out fracture includes the orbital floor and medial orbital wall. Other muscle entrapments would not produce difficulty with upward gaze. 2024
27
A 15-year-old boy is brought to the emergency department after he sustained facial injuries during a fall. CT scans are shown. The fractured line in the lateral orbit indicated by the star on the CT scans most likely involves which of the following craniofacial sutures? A) Frontosphenoid B) Zygomaticofrontal C) Zygomaticomaxillary D) Zygomaticosphenoid E) Zygomaticotemporal
The correct response is Option D. The fracture shown is a zygomaticomaxillary complex fracture, which involves fractures along the inferior orbital rim and floor, the zygomaticomaxillary region, the zygomatic arch, the zygomaticofrontal suture, and the zygomaticosphenoid suture along the lateral orbital wall. Reduction of the zygomaticosphenoid region provides excellent guidance about the overall three-dimensional reduction of the entire zygomatic complex fracture. While the zygomaticomaxillary, zygomaticotemporal, and zygomaticofrontal sutures are also involved in the zygomaticomaxillary complex, they are not labeled. The frontosphenoid suture (inferior-lateral cranium) is not involved in azygomaticomaxillary complex fracture. 2024
28
A 23-year-old woman is brought to the emergency department immediately after sustaining facial fractures in a motor vehicle collision. Physical examination shows right telecanthus with rounding of the medial canthal region. A right Markowitz-Manson type III naso-orbito-ethmoid fracture is suspected. Which of the following is the most appropriate technique to address this patient’s deformity? A) Closed reduction and splint immobilization B) Medial canthal reconstruction with transnasal wiring/suturing C) Open reduction and internal fixation of the central bony fragment of the naso-orbito-ethmoid D) Primary repair of the medial canthal tendon
The correct response is Option B. The Markowitz-Manson classification of naso-orbito-ethmoid fractures centers on the status of the medial canthal tendon and its bony attachments. In a type I fracture, the medial canthus is attached to the large central bony fragment. Open reduction and internal fixation of the fragment will address the resulting deformity. In a type II fracture, the bone is comminuted but the canthal tendon is still attached. Open reduction and internal fixation, and wire fixation with or without a bone graft can improve the deformity. In a type III fracture, the medial canthus has been completely avulsed and should be reconstructed with transnasal wiring. Closed reduction/splint immobilization, and primary repair have not shown efficacy in treating a type III naso-orbito-ethmoid fracture. 2024
29
A 3-year-old girl is brought to the emergency department after sustaining a dog bite to the lower lip. Physical examination shows the laceration extends through the mucosa, orbicularis oris muscle, and dry vermilion. Which of the following is the most appropriate suture material for repair of the mucosal layer? A) Braided polyester B) Chromic gut C) Nylon D) Polydioxanone E) Polypropylene
B Optimal suture material choice is essential for appropriate tissue approximation to maximize wound healing, minimize wound dehiscence, and create an ideal scar appearance. Ischemia, excess wound tension, and tissue injury should be minimized during wound closure. The surgeon should consider the smallest suture size that will accomplish these goals to minimize tissue trauma and foreign body burden within tissues. Monofilament sutures glide through tissue with lower tissue resistance as compared with multifilament sutures. Multifilament sutures possess higher tensile strength and flexibility but create greater tissue trauma. They also have increased suture sinus and infection risks. Chromic gut suture is a naturally derived monofilament suture made from chrome tanning solution-bathed mammalian collagen strands. It is an ideal choice for oral mucosal approximation in a child, since its tensile strength remains for 10 to 14 days. It is digested by endogenous enzymes and does not require subsequent suture removal, which can present challenges with a small child. Polydioxanone is a synthetic polyester polymer monofilament suture which retains 70% tensile strength at 2 weeks and 25% strength at 6 weeks. Absorption by hydrolysis is typically complete by 6 months. For oral mucosal repair, which heals rapidly, it is not an ideal choice since the sutures would remain in place as a foreign body much longer than is necessary. Nylon sutures are made from the first true synthetic suture fiber, with high tensile strength and low reactivity. Typical plastic surgery indications include skin approximation, vessel ligation, neurorrhaphy, microsurgical anastomosis, and oculoplastic surgeries. It is eventually encapsulated if left in place. Nylon sutures would require subsequent removal in a small childs mucosal repair, which could require another operative trip to accomplish safely. Polypropylene suture is a nonabsorbable synthetic monofilament. In facial laceration repairs, it is commonly utilized as a pull-through suture for cutaneous traumatic facial lacerations with a goal of ideal aesthetics and minimization of track marks. It would also require subsequent suture removal in a small child for a mucosal repair. Braided polyester (polyethylene terephthalate) suture is a nonabsorbable permanent multifilament suture typically used for ligament or tendon repair procedures. It is coated with polybutylate to allow smoother passage through tissue and easier knot-tying. As a permanent foreign body, it can cause infection, sinuses, and granulation tissue. It would also require subsequent suture removal when placed in the oral mucosa. 2024
30
Stensen’s duct can be found at which of the following anatomic sites? (A) At the mandibular angle (B) At the preauricular border (C) At the zygomatic arch (D) Between the superficial and deep lobes of the parotid gland (E) Within the buccal space
The correct response is Option E. Stensen’s duct can be found within the buccal space, which is bordered anteriorly by the orbicularis oris muscle, posteriorly by the edge of the masseter muscle, superiorly by the zygomaticus major muscle, and inferiorly by the fascial attachment of the buccinator muscle to the mandible. The duct develops deep within the parotid gland and emerges from the superior third of the gland at its anterior border, then courses below the zygomatic arch and enters the buccal space, inserting into the buccinator and then entering the oral cavity opposite the upper second molar. The facial artery and vein, buccal branches of the facial nerve, and buccal fat pad can also be found within the buccal space. Although Stensen’s duct and branches of the facial nerve are vulnerable to inadvertent dissection in the area of the parotid gland, there are no major arterial branches in this region. The external carotid artery can be found within the angle of the mandible, while the superficial temporal artery is located within the preauricular border. The transverse facial artery is found at the zygomatic arch. These locations do not lie within the typical course of Stensen’s duct.
31
A 45-year-old woman is evaluated because of a midline mass in the neck. History includes no prior neck surgeries. The patient reports no breathing, swallowing, or speaking problems. Physical examination shows a 2-cm mass that is mobile and nontender. The mass moves when she swallows and protrudes her tongue. Which of the following is the most appropriate surgical treatment? A) En bloc excision of the mass only B) En bloc excision with central hyoidectomy only C) En bloc excision, central hyoidectomy, and suprahyoid excision D) Incision and drainage
The correct response is Option C. A thyroglossal duct cyst is a lesion that occurs due to failure of the thyroglossal duct to obliterate during fetal development and is a remnant of the thyroid glands descent from the foramen cecum to the pretracheal space. It presents as a midline mass, moves with swallowing and tongue protrusion, has a bimodal presentation, and commonly presents early at age 6 years and later at age 45 years. Treatment is complete removal of the mass, including en bloc excision of the mass, central hyoidectomy, and excision including a core of tissue of the suprahyoid portion of the tract superior to the foramen cecum at the base of the tongue, known as the Sistrunk procedure. This results in the lowest rates of recurrence. Simple excision, even with en bloc removal, can result in recurrence rates of 55 to 70%, and if only excision of the lesion and central hyoidectomy are performed, the recurrence rate is 20%. Including a suprahyoid resection (Sistrunk procedure) decreases recurrence rates to 3 to 5%. Incision and drainage of a thyroglossal duct cyst would result in a high rate of recurrence and increase complications/recurrence for definitive excision of this lesion. Inferior margins are not indicated for removal of thyroglossal duct cysts. 2024
32
An 11-year-old boy is brought for evaluation because of a 3-day history of mild fever and sore throat. Physical examination shows a 2 × 4-cm enlarging midline neck mass that moves with swallowing; tongue protrusion is noted. CT scan shows a well-defined lesion with uniform fluid attenuation and peripheral rim enhancement. Which of the following is the most appropriate management at this time? A) Administration of systemic antibiotics B) Incision and drainage C) Marsupialization D) Sistrunk operation E) Observation only
The correct response is Option A. For this patient with signs and symptoms of an infected thyroglossal duct cyst, acute management with antibiotics is the recommended treatment. Broad-spectrum antibiotics targeted towards oral flora are appropriate and can be combined with needle aspiration of the fluid. Incision and drainage may be acceptable where abscess formation is seen but should be avoided in acute cases due to the risk for fistula formation. A Sistrunk operation is the definitive treatment, but it is not recommended in acute infection. Marsupialization is often not required with less invasive options. Observation only is not indicated. 2024
33
An 8-year-old boy is evaluated because of a 3-week history of a 2-cm soft mass of the midline neck. On examination, the mass is nontender and located slightly below the hyoid bone. There is no appreciated punctum in the skin. The mass elevates with protrusion of the tongue. Ultrasonography shows a cystic structure of the mass. Which of the following interventions is the most appropriate definitive treatment for this patient? A) CT-guided biopsy B) Incision and drainage C) Needle aspiration D) Sistrunk operation E) Thyroidectomy
The correct response is Option D. The diagnosis for this patient is a thyroglossal duct cyst. Thyroglossal duct cysts are one of the most common congenital neck masses and typically present in children within the first decade of life. These lesions are usually characterized as soft, nontender, midline swelling located at the level of the hyoid bone or adjacent. A defining characteristic is the elevation of the lesion when protruding the tongue due to the connection to the base of the tongue. The most widely accepted surgical intervention of thyroglossal duct cysts is the Sistrunk operation, which includes excision of the cyst, the middle part of the hyoid bone, and the surrounding tissue around the thyroglossal tract. The Sistrunk operation is associated with lower recurrence rates compared with cyst excision alone. Acutely infected thyroglossal duct cysts may benefit from incision and drainage along with antibiotics to temporize the lesion until definitive treatment. Needle aspiration could potentially be used as a diagnostic procedure but not as the definitive treatment. Thyroidectomy is not indicated in this patient because thyroglossal duct cysts arise from the remnants of the thyroglossal duct and not the thyroid itself. CT-guided biopsy is not needed to establish diagnosis and is not adequate treatment. 2024 
34
Which of the following structures is derived from the same pharyngeal arch as the levator veli palatini? A) Common carotid artery B) Ductus arteriosus C) Malleus bone D) Trigeminal nerve E) Vagus nerve
The correct response is Option E. The levator veli palatini is derived from the fourth pharyngeal arch during embryologic development, along with the uvula, palatopharyngeus muscle, palatoglossus muscle, pharyngeal constrictors, cricothyroid, salpingopharyngeus, laryngeal cartilages, vagus (X) nerve, aortic arch, and right subclavian artery. The common carotid artery is derived from the third pharyngeal arch, along with the stylopharyngeus muscle, glossopharyngeal (IX) nerve, and portions of the hyoid bone (greater cornu and lower body). The ductus arteriosus is derived from the sixth pharyngeal arch, along with the arytenoid muscles, epiglottis, aorta, and pulmonary artery. The malleus bone and mylohyoid are derived from the first pharyngeal arch, along with the anterior digastric muscle, tensor veli palatini muscle, squamous portion of the temporal bone, zygoma, maxilla, premaxilla, Meckel cartilage, trigeminal (V) nerve, and first aortic arch artery. 2024
35
A 10-year-old boy with unilateral lobular-type microtia undergoes first-stage auricular reconstruction using a Brent technique. Ten days postoperatively, the patient is brought for follow-up evaluation. Examination shows a 3 × 5-cm eschar over the superior helix, antihelix, concha cymba, and upper tragus. Which of the following is the most appropriate management? A) Daily hyperbaric oxygen therapy for 3 weeks B) Debridement of the eschar and closure with a mastoid fascia flap and skin graft C) Debridement of the eschar and closure with a temporoparietal fascia flap and skin graft D) Debridement of the eschar and primary closure E) Topical sulfamylon and twice weekly follow-up evaluation
The correct response is Option C. Soft-tissue coverage and treatment of related complications is one of the most challenging aspects of auricular reconstruction. The clinical scenario describes a large area of soft-tissue compromise of the superior and anterior aspect of the auricular construct. Conservative management of areas of cartilage exposure, such as with topical sulfamylon, is appropriate in small defects less than 1 cm2 in size. The clinical scenario described is much larger than that with involvement of multiple anatomic subunits. Adequate debridement and coverage with vascular tissue is needed to limit the risk for infection and reabsorption of the cartilage grafts that may lead to significant loss of structure and definition. Similarly, hyperbaric oxygen treatment is a helpful adjunct to treat threatened soft-tissue flaps early after surgery to minimize tissue loss. In the described scenario, 10 days have passed since surgery and the formation of eschar implies tissue necrosis that is no longer salvageable. Local transposition flaps may be used to cover small soft-tissue defects in the anterior ear by taking advantage of soft-tissue laxity. Given the size of the defect and superior/anterior location of the defect, a local transposition flap would likely be under undue tension and include hair-bearing skin. Coverage of posterior construct structures with mastoid fascia flaps and a skin graft is an acceptable approach, but this defect is located anteriorly and superiorly, where a temporoparietal fascia flap would provide ideal coverage. 2024
36
A 24-year-old man is evaluated because he is dissatisfied with the appearance of his ears. Physical examination shows underdevelopment of the antihelical fold with no conchal bowl hypertrophy. The lobule appears to not protrude. Suture otoplasty is planned. Which of the following suture techniques is most appropriate to address the antihelical fold in this patient? A) Furnas B) Mustardé C) Siegert D) Spira E) Stenström
The correct response is Option B. The Mustardé suture technique is a technique that recreates the antihelical fold with mattress sutures between the scaphoid and conchal fossa and without scoring the cartilage. One can consider the prominent ear as being comprised of varying contributions from three sources: underdeveloped antihelical fold, conchal hypertrophy, and lobule malposition. While scoring techniques were popularized in the 1960s and 1970s, more recent concerns about the integrity of the cartilage and risk for infection have led to surgeons favoring suture techniques. Suture techniques, with or without resection of the lobule or conchal bowl, can address all three aspects of the prominent ear. The Furnas technique uses sutures to fix the conchal bowl to the mastoid periosteum to address a hypertrophic conchal bowl. The Spira technique combines a wedge excision of the lobule with a deep dermal-to-periosteum suture to reposition the lobule, and it also uses a conchamastoid suture to address a hypertrophic conchal bowl. The Stenström technique is not a suture technique; it relies on abrasion of the anterior surface of the antihelix, which induces bending of the cartilage away from the disruption. The Siegert technique is used for lobuloplasty. 2024
37
A boy with congenital microtia is scheduled to undergo total ear reconstruction. Compared with autologous reconstruction with a rib cartilage framework, alloplastic reconstruction with a porous polyethylene framework is most likely to provide which of the following advantages? A) Can be performed in younger patients B) Decreased infection rates C) Decreased postoperative exposure rates D) More easily used in a previously irradiated bed
The correct response is Option A. Recent systematic reviews suggest that alloplastic ear reconstruction has an increase in complications when performed in irradiated beds, increased exposure and extrusion rates, and increased infection rates. Reported aesthetic outcomes did not seem to be different among the different techniques. Potential advantages of alloplastic prostheses include fewer stages of surgery and the ability to be performed in patients of a younger age, as young as age 4 to 5 years (compared with autologous reconstruction, which has more stages of surgery and is preferably performed in older children, with earlier techniques suggesting age 7 to 9 years, and now preferably older at age 9 to 11 years with more rib availability).
38
A 30-year-old woman who underwent Mohs micrographic surgery for basal cell carcinoma of the ear comes to the clinic because of an ear defect. On physical examination, surgical margins are negative for malignancy. A photograph is shown. Which of the following reconstruction methods will most likely have the most durable and best aesthetic result in this patient? -pic of top part of helix missing- A) Bilobe flap B) Chondrocutaneous advancement flap D) Composite grafting E) Full-thickness skin grafting F) Scapha wedge excision with primary closure
The correct response is Option B. The chondrocutaneous advancement flap, or Antia-Buch flap, is extremely helpful in reconstruction of small- to medium-sized upper third helical defects. While such a reconstruction may make a slightly smaller ear, it is usually well tolerated. A postoperative photograph of the patient in this scenario is shown. Scapha wedge excision with primary closure and full-thickness skin grafting would leave an unacceptable standing deformity. Bilobe flaps are of limited utility in the upper third of the ear. Composite grafts may be useful for smaller helical defects, especially in younger patients; however, in defects of this size, the success rate is poor. 2024
39
An otherwise healthy 11-year-old boy is brought to the emergency department approximately 30 minutes after sustaining a dog bite to the left ear. Physical examination shows a partial (30%) ear amputation, consisting of a 2 × 4-cm, full-thickness fragment of the helix, antihelix, and conchal bowl. The amputated fragment is intact and appropriately preserved on ice. The patient is brought urgently to the operating room for irrigation, debridement, and attempted replantation. A branch of the posterior auricular artery is found on the amputated fragment, and arterial inflow is restored with primary anastomosis. However, no vein target is identified on the fragment. Which of the following is the most appropriate next step in management? A) Completion of inset of the replanted fragment and leech therapy B) Debridement of the amputated fragment and immediate reconstruction with a local flap C) Debridement of the amputated fragment and immediate reconstruction with a skin graft D) Debridement of the amputated fragment and local wound care E) Harvest of an appropriately sized vein for interposition vein grafting
A Although venous repair should be attempted whenever possible, it is not totally necessary to primarily restore venous outflow. In the reported cases of complete or partial ear replantation, approximately one-third report restoration of arterial inflow without any form of venous reconstruction, but replant survival rate is not significantly changed by venous repair (64.7% success in the “no venous repair” cohort versus 62.9% in the “venous repair” cohort). Successful venous repair may decrease the transfusion requirement for patients undergoing leech therapy, but it does not affect replant survival rates. Discarding the replanted fragment would result in additional procedures, further morbidity, or a suboptimal aesthetic outcome. There was no identifiable vein on the fragment; without a recipient vein, a vein graft provides no benefit. Completion of inset of the replanted ear fragment and expectant management with leeches and permissive bleeding is the only option that preserves the possibility of a successful replantation while also leaving reconstructive doors open in the event of an eventual replant failure. 2024
40
A 13-month-old male infant is brought to the office for evaluation of a mass on the right lower leg. The mass was present at birth, but during the past 2 weeks, it has grown and become more edematous, firm, and purple. Physical examination shows cutaneous petechiae at multiple sites including the abdomen and extremities. Photographs of the mass are shown. Results of laboratory studies show profound thrombocytopenia and hypofibrinogenemia with an increased D-dimer concentration. Which of the following is the most likely diagnosis? A) Angiolipoma B) Hemangioma C) Kaposiform hemangioendothelioma D) Liposarcoma
The correct response is Option C. This patient is presenting with a congenital mass that has recently enlarged and become more discolored. The findings are consistent with Kasabach-Merritt syndrome (KMS), ruling out the other tumors and vascular malformations. KMS does not occur with hemangioma, a true vascular tumor, as originally proposed. Rather KMS occurs with two distinct but related tumors: kaposiform hemangioendothelioma (KHE) and tufted angioma (TA). KHE involves a spectrum of lesions from small, superficial tumors to large, infiltrative lesions with potentially life-threatening complications as seen in patients with KMS. The majority of patients present in infancy with a classic cutaneous vascular lesion; increasing discoloration of the skin involved and progressive enlargement of the tumor frequently occurs. KHE may have a similar appearance to a hemangioma; however, hemangiomas traditionally proliferate and then involute over the course of several years. KMS is a life-threatening, consumptive coagulopathy most frequently associated with an underlying vascular tumor such as KHE or TA as described previously. KMS is characterized by severe thrombocytopenia, microangiopathic anemia, hypofibrinogenemia, and increased fibrin split products in the presence of a rapidly enlarging tumor. Patients with KMS may have disseminated cutaneous petechiae. A deep red-purple color with advancing ecchymotic rim is often seen on the skin overlying these invasive vascular tumors. The tumors with KMS are described as warm, leathery, and nodular to palpation. Vascular malformations such as venous malformations (VMs) are present at birth and grow commensurately with the patient. They may not be clinically apparent at birth until a period of growth such as in early childhood. Vascular malformations may have rapid growth subsequent to trauma, infection, or hormonal changes such as puberty. VMs are not associated with KMS. VMs can be associated with localized intravascular coagulation (LIC), especially extensive/large complex VMS characterized by decreased fibrinogen, increased D-dimers, and soluble fibrin complex; however, platelet counts were normal or mildly decreased. VM-associated LIC can be a lifelong coagulopathy, whereas KMS is associated with platelet trapping associated with distinct vascular tumors (KHE and TA) in infancy. Treatment is very different between the two clinical entities. The mass has been present since birth, so liposarcoma is very unlikely. Angiolipomas are benign masses of adipocyte origin; these often present with adult age, and there are often multiple angiolipomas present. Angiolipomas are not associated with KMS. 2024
41
A 16-year-old girl is brought for evaluation of multiple cartilaginous growths over the extremities. X-ray studies show the appearance of enchondromatosis. Which of the following additional findings is most consistent with a diagnosis of Maffucci syndrome rather than Ollier disease in this patient? A) Limb-length discrepancy B) Secondary chondrosarcoma C) Somatic mosaic mutations in IDH1/IDH2 D) Unilateral enchondromas E) Venous malformation
The correct response is Option E. This patient presents with multiple enchondromatosis, with the differential diagnoses being Ollier disease and Maffucci syndrome. Maffucci syndrome has multiple cutaneous and visceral cavernous hemangiomas (venous malformations). Ollier disease presents similarly without venous malformations. Both Maffucci syndrome and Ollier disease may present with secondary malignant transformation to chondrosarcoma, somatic mosaic mutations (generally IDH1 and IDH2), similar incidences of unilateral presentation, and limb-length discrepancy. 2024
42
A 1-week-old male newborn is evaluated at bedside because of a patchy discoloration overlying the left lower extremity. The patient’s parents also have noticed asymmetry of the lower limbs. A photograph is shown. MRI of the left lower extremity shows an underlying venous and lymphatic malformation. Which of the following is the most likely diagnosis? A) Klippel-Trénaunay syndrome B) Parkes-Weber syndrome C) Rendu-Osler-Weber syndrome D) Sturge-Weber syndrome
The correct response is Option A. Capillary malformations/port-wine stains affect 0.3% of newborns and are most frequently found on the face. They can exist anywhere on the body and incidence has a female-to-male ratio of 3:1. Klippel-Trénaunay syndrome is associated with a patchy port-wine stain on an extremity overlying a deeper venous and lymphatic malformation with associated skeletal hypertrophy. Parkes-Weber syndrome is similar to Klippel-Trénaunay syndrome, but it is distinguished by the presence of an arteriovenous fistula on MRI. Sturge-Weber syndrome is associated with a large facial port-wine stain involving the V1 and V2 trigeminal (V) nerve distribution. This is usually associated with leptomeningeal venous malformations and frequent mental retardation. Telangiectases of the skin and mucous membranes, epistaxis, and a positive family history make up the classic triad of Rendu-Osler-Weber syndrome. 2024
43
A 6-month-old male infant with cervicofacial lymphatic malformation diagnosed prenatally is brought for evaluation because of a sudden enlargement of the lymphatic malformation. A photograph from the newborn period is shown. Physical examination today shows erythema and pain over the lesion; there is no fever. In addition to admission to the hospital, which of the following is the most appropriate next step in management? -Large neck swelling/mass- A) Emergent decompression B) Emergent sclerotherapy C) Initiation of sirolimus D) Intravenous administration of antibiotics E) Pain control with intravenous acetaminophen
The correct response is Option D. Lymphatic malformations are at risk for infections, which present as sudden enlargement, erythema, pain, and leukocytosis (or left shift), with or without fever. Any sudden enlargement should be treated as infection unless proven otherwise. In the cervicofacial area, the sudden enlargement could put the airway at risk and, even with a tracheostomy, the patient should be admitted. Intravenous antibiotics are the treatment of choice. Decompression will result in rapid reaccumulation of fluid. There is no indication for sclerotherapy when the lesion is actively infected and may cause seeding/skin infection/breakdown. Sirolimus should be held when there is an active infection, so this is definitely not the time to start if the patient is not already on it. The patient needs to be treated with antibiotics, and pain control treatment alone may miss an infection that can progress to sepsis quickly. 2024
44
An otherwise healthy 6-month-old male infant is brought to the clinic by his parents because of a mass on the left cheek at the lower eyelid-cheek junction. A flat, red skin lesion was present at birth. At 2 months of age, the lesion began growing rapidly and continues to grow. The parents report there have been no episodes of the lesion bleeding. Physical examination today shows a 2-cm elevated, well-defined, mobile red mass that is not tender. Ophthalmologic evaluation shows no obstruction to the visual axis. Which of the following is the most appropriate next step in management? A) Corticosteroid therapy B) Propranolol therapy C) Serial pulsed-dye laser treatments D) Surgical excision E) Observation
B Infantile hemangioma is the most common tumor in the pediatric patient. The typical presentation is rapid growth in the first months of life followed by an involution process over years. Pediatric patients are often brought for consultation with the plastic surgeon for lesions on the face, and it is imperative that surgeons are familiar with the management strategies for these lesions (Rodríguez et al, Sebaratnam et al). Of particular concern are lesions near the visual axis, as obstruction of vision in infancy may lead to impaired development of the visual cortex. This patient has a facial infantile hemangioma that is nonulcerative (as it has not bled), nonpainful, and nonobstructing of vision. First-line management of continually growing lesions is medical therapy rather than surgical excision (Chinnadurai et al). Propranolol has emerged as the first option because it has demonstrated greater success with fewer side effects compared with oral corticosteroids (Izadpanah et al, Al-Haddad et al). Pulsed-dye laser treatment has demonstrated the most success in flat lesions typical of capillary malformations and requires a general anesthetic in the infant population to successfully perform. Surgical excision would be considered either with failure of propranolol or progression of the lesion to obstruct the visual axis. Observation is not ideal as this lesion may interfere with the development of the visual axis if left alone, as it is likely to rapidly grow in size. Therefore, the best initial option in this scenario is oral propranolol. 2024
45
A 55-year-old man undergoes a modified radical neck dissection for squamous cell carcinoma of the tongue. During the procedure, inadvertent injury of the nerve that runs obliquely through the posterior triangle of the neck is most likely to result in which of the following? A) Decreased sensation of the tongue B) Decreased shoulder mobility C) Hoarseness D) Weakened neck rotation E) Weakened tongue movement
The correct response is Option B. The spinal accessory nerve, though spared during modified radical neck dissection, is at risk for injury since it runs in the posterior triangle of the neck. The spinal accessory nerve contains motor fibers to the sternocleidomastoid and trapezius muscles. The nerve passes through or posterior to the sternocleidomastoid muscle and then travels obliquely until it pierces the trapezius muscle, approximately 2 to 4 cm above the clavicle. Injury to the spinal accessory nerve in the posterior triangle results in shoulder drooping or decreased mobility, adhesive capsulitis of the glenohumeral joint, and aberrant scapular rotation. If the spinal accessory nerve is damaged within the posterior triangle of the neck, sternocleidomastoid muscle function is spared as the nerve has already given its motor branches to this muscle more proximally in the neck. In this scenario, weakened neck rotation would not be expected. Weakened tongue movement is consistent with injury to the hypoglossal nerve. Decreased sensation of the tongue is consistent with injury to the lingual nerve. Hoarseness would likely result from vocal cord paralysis after injury to the recurrent laryngeal branch of the vagus nerve. 2024
46
A 26-year-old man is evaluated for numbness at the most anterior region of the hard palate behind the upper incisors and the posteroinferior portion of the nasal septum. Which of the following nerves is most likely involved? A) Anterior ethmoidal B) Buccal C) Greater palatine D) Lesser palatine E) Nasopalatine
The correct response is Option E. The nasopalatine nerve is a branch of the maxillary nerve (cranial nerve V2) and travels through the incisive foramen. It innervates the anterior hard palate behind the incisive foramen and the internal nasal cavity. The nasopalatine nerve travels from the pterygopalatine ganglion across the nasal septum towards the incisive canal. The anterior ethmoidal nerve is a branch of the ophthalmic nerve (cranial nerve V1) and supplies sensation to the roof of the nasal cavity, anterior nasal septum, anterior and middle ethmoid air cells, and parts of the meninges. It is involved in the diving reflex. The greater palatine nerve carries both sensory fibers from the maxillary nerve and parasympathetic fibers from the nerve of the pterygoid canal (vidian nerve). It supplies sensation to the gums, mucous membrane, and glands of the hard palate. The lesser palatine nerve is a branch of the maxillary nerve (cranial nerve V2). It descends through the greater palatine canal alongside the greater palatine nerve, emerges separately through the lesser palatine foramen posteriorly, and supplies sensation to the soft palate, tonsil, and uvula. The buccal nerve is the only sensory branch of the anterior mandibular division of the trigeminal nerve. It innervates the major part of the buccal mucosa, the inferior buccal gingiva in the molar area, and the skin above the anterior part of the buccinator muscle. 2024
47
The cosmetic result of a 1.5-cm full-thickness skin nasal defect allowed to heal by secondary intention is most acceptable in which of the following locations? (A) Alar margin (B) Central nasal tip (C) Dorsal bridge (D) Medial canthal area (E) Soft triangle
The correct response is Option D. Healing by secondary intention is most acceptable for nasal defects involving the medial canthal area. Although spontaneous healing is mostly overlooked in the management of nasal defects, it should be a consideration in patients with concomitant medical conditions or previous radiation therapy, or in the management of those patients who have developed infection following Mohs’ surgery or who refuse to undergo surgery. According to one study of 282 patients, the size and location of the nasal defect best predicted the cosmetic outcome. Defects of the medial canthal area, glabella, philtrum, and nasolabial fold showed good cosmetic results in more than 90% of patients who underwent healing by secondary intention; in contrast, defects of the ala, rim, soft triangle, and nasal tip showed the greatest contracture and rim distortion when allowed to heal by secondary intention. Large defects, involving one subunit, also healed unacceptably. Defects of the nasal dorsum and sidewall had a moderate acceptability rate of 70% to 80%; depressed scars and distortion of the cheek groove were the most commonly sited adverse sequelae. Another study results.
48
The dorsal nasal flap is most appropriate for coverage of which of the following defects of the nose? (A) A 1-cm defect of the alar base (B) A 1-cm defect of the columella (C) A 2-cm defect of the medial canthus (D) A 2-cm defect of the nasal tip (E) A 3-cm defect of the lateral wall
The correct response is Option D. The dorsal nasal flap was first described in 1967. Flap transfer usually involves rotation and caudal advancement of the entire skin of the nasal dorsum and the glabella. It also can be accomplished in a single-stage procedure while the patient is receiving local anesthesia. Since its introduction, the dorsal nasal flap has been modified by many surgeons. For example, the pedicle can be back-cut to the angular artery, and the glabellar portion of the flap need not be used. The dorsal nasal flap provides an excellent color, texture, and thickness match, which is its greatest advantage. It is predominantly used to cover defects that occur following excision of lesions of the nasal tip. Defects as large as 2 cm may be covered with this flap. A potential disadvantage associated with use of this flap is the violation of other aesthetic subunits of the nose. A dorsal nasal flap will not reach a columellar defect. A medial canthal defect is easily reconstructed using a skin graft or small local flap. A nasolabial flap is best used for coverage of a defect of the alar base. A 3-cm defect is beyond the limits of a dorsal nasal flap; a flap that provides additional tissue (such as a forehead flap) would likely be needed to close this defect.
49
Which of the following flaps is most appropriate for coverage of a 2-cm full-thickness skin defect of the columella? (A) Bi-lobe flap (B) Glabellar flap (C) Median forehead flap (D) Nasolabial flap (E) Scalping flap
The correct response is Option D. The nasolabial flap should be used for coverage of a 2-cm full-thickness skin defect of the columella. This flap, which is based on the angular artery (terminal branch of the facial artery), can be tunneled deeply to provide tissue for intraoral or columellar reconstruction. It can also be used to cover defects of the lower nose, nasal alae, and upper lip. Bi-lobe flaps are best used for coverage of defects involving the upper and middle thirds of the nose, not the columella and nasal tip. Glabellar flaps are used for coverage of defects involving the medial canthal and upper nasal regions. The median forehead flap, which is based on the supratrochlear artery, is more useful for coverage of large defects of the nose. A scalping flap is appropriate for near-total and total nasal reconstruction.
50
A 25-year-old man was assaulted 72 hours ago and presents with a noncomminuted, minimally displaced right angle fracture of the mandible. The plastic surgeon plans to perform open reduction and internal fixation. Which of the following factors is most likely to place the patient at greatest risk for a postoperative complication? A) Perioperative ampicillin-sulbactam administration B) Placement of two mini-plates C) Postoperative maxillomandibular fixation D) Sex of patient E) Time to operative intervention
The correct response is Option B. In multiple studies, the use of a single plate along either the oblique ridge or the lateral cortex demonstrates the lowest incidence of complications for this type of fracture. Specifically, placement of two mini-plates has been associated with an increased risk for postoperative complication. Postoperative maxillomandibular fixation has not been shown to influence the incidence of complications. Furthermore, time to operative intervention and the sex of the patient have not been shown to increase the complication rate. Factors such as smoking and the number of fractures have been associated with an increased risk for complication with mandible fractures. Finally, perioperative ampicillin-sulbactam administration has been shown to decrease the risk for complications in this patient population.
51
A 2-month-old male infant is brought to the office for evaluation of an abnormal head shape. The patient is a twin who was born at 33 weeks’ gestation, and he was in the intensive care unit for 3 weeks. History is otherwise unremarkable. Physical examination shows flattening of the right posterior skull, left occipital protuberance, and right ear anterior displacement. There is decreased neck mobility. Which of the following is the most appropriate next step in management? A) Cranial vault remodeling B) CT scan C) Helmet orthosis D) MRI of the brain E) Physical therapy
The correct response is Option E. This patient presents with posterior positional plagiocephaly, which is commonly seen in male patients, first-born patients, multiple births, and prolonged immobilization. The first step would be to actively observe, which means educating the caregivers on repositioning and encouraging tummy time. Patients benefit from a physical therapy evaluation and possible treatment of torticollis. If there is no improvement, patients should be evaluated for helmet orthosis. There is no need for imaging or surgical consultation or intervention. Although this patient may eventually require a helmet orthosis, the FDA approval for helmeting orthosis is ages 3 to 18 months. 2024
52
A 2-week-old male infant diagnosed with severe micrognathia is intubated in the neonatal intensive care unit for severe airway obstruction. Mandibular distraction osteogenesis with a 25-mm movement goal to improve the airway is planned. Which of the following distraction phase durations is most appropriate in this neonatal patient? A) 1-week total activation phase B) 2-week consolidation phase before removal of hardware C) 2-week latency phase after initial osteotomy D) 3-week total activation phase E) 4-week latency phase after initial osteotomy
The correct response is Option D. The treatment of Pierre Robin sequence (retrognathia, glossoptosis, and airway obstruction) has been significantly improved with distraction osteogenesis. Distraction osteogenesis is a technique to generate new bone by manipulating surgically osteotomized bone with a device to control the rate and vector of the movement. Beyond the osteotomy, there are three phases to distraction osteogenesis: latency, activation (distraction), and consolidation. The latency phase is a time period that is required for callus formation. The latency period should be short enough to avoid calcification and premature union, but long enough to allow for adequate callus formation. The widely accepted duration of latency is less than 1 week; however, for neonates the range is often 1 day. Two-week and 4-week latency phases are too long and increase risk for premature union. The activation phase is the expansion of the mechanical device by serial axial screw movements to lengthen the formed callus before calcification. The rate and frequency of the activation is important for successful distraction. Increased rate/frequency can result in ischemia at the cellular level and lead to malunion/nonunion. Decreased rate/frequency can result in premature ossification. The frequency is typically two to four times a day with a 1- to 1.5-mm activation rate per day. For a 25-mm movement, the activation phase will take 3 weeks. Reaching 25-mm movement in 1 week will require a 3+-mm activation rate per day and will likely result in nonunion. The consolidation phase is a long period of immobilization to allow the expanded callus to remodel into lamellar bone, avoiding a cartilaginous intermediate. The time range for consolidation is variable depending on the location of the distraction site and rate of bone metabolism; however, the average is 8 weeks. It is recommended that the consolidation phase is kept twice as long as the activation phase. A 2-week consolidation would be too abbreviated. 2024
53
Posterior cranial vault expansion by distraction osteogenesis as the initial treatment of craniosynostosis is most appropriate in which of the following settings? A) Nonsyndromic sagittal suture craniosynostosis B) Severe trigonocephaly metopic synostosis C) Syndromic bicoronal craniosynostosis D) Unilateral lambdoid craniosynostosis
The correct response is Option C. Multisuture synostosis increases the risk for intracranial pressure and more severe skull deformity. Different institutions follow different successful protocols to manage cranial vault expansion to treat bicoronal synostosis. Strip craniectomy, fronto-orbital advancement, and posterior cranial vault expansion have all been described as first procedures. Posterior cranial vault expansion has been shown to allow for more expansion than fronto-orbital advancement. When a patient has syndromic turribrachycephaly, posterior cranial vault expansion with distraction is performed to garner more expansion as a first procedure. The forehead appearance and shape is best corrected with fronto-orbital advancement instead of posterior cranial vault expansion. Sagittal synostosis may be addressed with strip craniectomy and helmet treatment at an early age or with an open modified pi procedure at an older age in infancy. Posterior expansion would increase dolichocephaly and would not be indicated. Trigonocephaly is best addressed with an anterior procedure such as fronto-orbital advancement. A unilateral lambdoid suture could be addressed with suturectomy and helmet, if identified early, or switch cranioplasty posteriorly. A bilateral posterior distraction would be less likely to improve shape. 2024
54
A 3-year-old girl undergoes resection of fibrous dysplasia in the left orbital region. A preoperative CT scan and intraoperative photograph are shown. Which of the following skeletal reconstruction materials is most likely to result in the lowest overall risk for reoperation? A) Autologous bone B) Demineralized bone matrix and resorbable mesh C) Methyl methacrylate implant D) Polyetheretherketone implant E) Titanium mesh
The correct response is Option A. Autologous bone is the most appropriate material for cranial reconstruction in a growing child. There are many advantages over alloplastic materials including decreased cost, lower infection risk, ability to grow along with the cranium, and greater long-term stability due to osseointegration. Most studies show that the risk for reoperation using autologous bone in children is very low, especially in very young children (like the one in this scenario) who have excellent bone-healing capabilities. While implant technology has improved, none of the synthetic materials listed (polyetheretherketone, methyl methacrylate, or titanium mesh) osseointegrate or grow along with the surrounding cranium. Thus, implant stability is only as good as the peripheral fixation (small plates and screws), and in very young children, continued cranial growth can and often does lead to gradual implant destabilization and the need to replace the implant. The revision rate with demineralized bone and resorbable mesh reconstruction is unacceptably high. 2024
55
Which of the following anomalies is most commonly seen in patients with nonsyndromic unicoronal craniosynostosis? A) Cataract B) Coloboma C) Keratopathy D) Ptosis E) Strabismus
The correct response is Option E. Ophthalmologic pathology is often found in patients with craniofacial disorders. Of the findings listed, strabismus is most associated with nonsyndromic unicoronal craniosynostosis. Saethre-Chotzen syndrome can result in unicoronal craniosynostosis or bicoronal synostosis and is associated with lid ptosis. Exposure keratopathy can occur in patients with severe syndromic craniosynostosis (Apert, Crouzon, and Pfeiffer syndromes), most often in patients with bicoronal synostosis and mid face hypoplasia (ie, exorbitism). Cataract and coloboma can occur with many craniofacial syndromes, though they are not closely associated with nonsyndromic craniosynostosis. 2024
56
A 1-week-old female newborn is scheduled to undergo mandible distraction for Pierre Robin sequence. Which of the following types of teeth is most likely to be injured during this procedure? A) Ankylosed B) Deciduous C) Impacted D) Permanent E) Succedaneous
The correct response is Option D. The succedaneous teeth are the permanent teeth that replace the deciduous teeth. Permanent molars are not succedaneous teeth because they do not replace any primary teeth. Succedaneous teeth originate from successional laminae, whereas permanent molars originate from the general dental laminae. The most commonly injured teeth with mandible distraction using an inverted L osteotomy are the first and second molars due to their proximity to the osteotomy. The first molar bell caps are typically visible at birth. Impacted and ankylosed teeth typically occur later in development after the eruption of adjacent teeth that prevent or obstruct eruption of other teeth. 2024
57
A 3-year-old girl is evaluated because of bilateral facial malformations that include symmetrical midfacial hypoplasia, microtia, downward slanting palpebral fissures, lower eyelid coloboma, and micrognathia. Genetic testing is positive for a mutation in TCOF1. CT scans are shown. Which of the following is the most likely diagnosis? A) Craniofacial microsomia B) Kabuki syndrome C) Pierre Robin sequence D) Saethre-Chotzen syndrome E) Treacher Collins syndrome
The correct response is Option E. The condition described is Treacher Collins syndrome. The classical clinical features include bilateral mid face hypoplasia, downward slanting palpebral fissures, lower eyelid coloboma, and micro/retrognathia. Patients most commonly have mutations in TCOF1, but less common mutations in POLR1C and POLR1D have also been described. While this condition can be associated with Pierre Robin sequence, the constellation of micro- or retrognathia, glossoptosis, and respiratory distress, the latter two findings are not part of this scenario. Craniofacial microsomia, also termed hemifacial microsomia, can occur bilaterally but is rarely symmetrical and not associated with mutations in TCOF1. Saethre-Chotzen syndrome is a craniofacial syndrome associated with mutations in TWIST1. Patients typically present with bilateral or unilateral coronal suture fusion and eyelid ptosis, but microtia or micrognathia are not part of the syndrome. Kabuki syndrome manifests with high arched eyebrows, long palpebral fissures, and growth and developmental delays. 2024
58
A 25-year-old woman is undergoing evaluation of microgenia. Cephalometric analysis shows marked bony deficiencies in both the anteroposterior and vertical dimensions. Occlusion is normal. Which of the following is the most appropriate management? (A) Chin implantation (B) Intraoral vertical osteotomy (C) Sagittal split osteotomy (D) Osseous genioplasty (E) Distraction osteogenesis
The correct response is Option D. In this patient who has microgenia with anteroposterior and vertical deficiencies of the chin, the most appropriate management is osseous genioplasty. Microgenia is one of several terms used to describe abnormalities of the chin. In this condition, the chin is small and bone is deficient in all three planes. Macrogenia describes a large chin; both macrogenia and microgenia can be associated with normal occlusion or mandibular prognathism. In patients with retrogenia, the chin is positioned posteriorly but is not necessarily small. Occlusion is normal in pure retrogenia. Patients with mandibular retrognathia have secondary retrogenia. Any of the above conditions can be associated with vertical abnormalities and chin asymmetry. This patient has moderate-to-severe vertical microgenia, which frequently includes anteroposterior deficiency of the chin. This results in decreased height of the lower face, creating an imbalance between the midface and lower face. This abnormality can be corrected by performing osseous genioplasty, which will increase the projection of the mental symphysis within the sagittal plane while compensating for the vertical deficiency. Chin implantation is recommended to increase the anteroposterior projection of the chin but does not correct vertical deficiency or excess. Both intraoral vertical osteotomy and sagittal split osteotomy of the mandibular ramus will alter dental occlusion and are not indicated for correction of vertical or horizontal deficiencies, especially in a patient who has normal occlusion. Distraction osteogenesis is indicated for treatment of skeletal deficiencies (ie, Pierre Robin sequence) but is an unnecessary, excessive option for correction of this deformity.
59
Which of the following mechanisms of action of the mandible occurs during the terminal 4 cm to 5 cm of jaw opening? (A) Rotation within the lower joint space (B) Rotation within the upper joint space (C) Translation and rotation within the lower joint space (D) Translation within the lower joint space (E) Translation within the upper joint space
The correct response is Option E. Normal opening of the mandible results from the synchronized movements of muscles surrounding the joint space. The articular disk separates the joint space into upper and lower spaces. The combination of motions of the mandibular condyle generates the motion of the temporomandibular joint. At rest and during rotation, the mandibular condyle is located in the lower joint space. During translation, the condyle moves into the upper joint space. Most patients have a maximal incisal opening of 4 cm to 5 cm. The initial 1 cm to 2 cm of jaw opening involves rotatory, or hinge, movements. Jaw opening at 2 cm to 3 cm is a combination of rotation and translation. The terminal 3 cm to 5 cm of jaw opening involves translatory movements only.
60
A 21-year-old man sustains blunt trauma to the face while playing football. On examination, he has unilateral pain and facial swelling; he is unable to open his mouth. Radiographs show a nondisplaced coronoid fracture. Which of the following is the most appropriate initial step in management? (A) Coronoidectomy (B) Maxillomandibular fixation (C) Endoscopic reduction and fixation (D) Open reduction and rigid internal fixation (E) Open reduction and wire fixation
The correct response is Option B. In this 21-year-old man who has an isolated nondisplaced fracture of the coronoid process, the most appropriate management is short-term maxillomandibular fixation. The tendons of the temporalis muscle act as splints for the fracture fragments, allowing osseous union to occur spontaneously. Because ankylosis may develop between the coronoid process and the zygomatic arch if immobilization is prolonged, fixation should only be applied for one to two weeks. Following removal of fixation, physical therapy may be required to re-establish the normal vertical dimension of the face. In rare patients who have displaced fracture fragments that obstruct normal mandibular motion, partial coronoidectomy may be performed. Endoscopic approaches to the temporomandibular joint are reserved for intra-articular injuries such as meniscal tears. Rigid or wire fixation is not required because of the expected osseous union associated with this type of fracture
61
A 30-year-old man sustains an open fracture of the right mandibular angle when he is ejected from an automobile during a motor vehicle collision. During surgical fixation, a fracture of the second molar of the mandibular arch on the right side is noted, and the tooth is removed. According to the Universal Tooth Designation System, this tooth is identified by which of the following numbers? A) 2 B) 15 C) 18 D) 31
The correct response is Option D. There are several systems available for tooth designation and numbering. In the United States, the most common system is the Universal/National Tooth Designation System, and it has been used by the American Dental Association since 1994. In this system, permanent teeth are numbered from 1 to 32, starting with the third maxillary molar on the right side, following counterclockwise around the arch to the third maxillary molar on the left side, descending to the third mandibular molar on the left side, and returning clockwise around the arch to the third mandibular molar on the right side. Under this classification, the second mandibular molar on the right side is tooth 31. Tooth 2 is the second maxillary molar on the right side. Tooth 15 is the second maxillary molar on the left side. Tooth 18 is the second mandibular molar on the left side. 2024
62
A 65-year-old man is brought to the emergency department after he sustained mandibular symphyseal, right body, and left angle fractures. At the time of admission, he is placed on preoperative ampicillin-sulbactam. Seven days after the initial injury and following stabilization, the patient undergoes open reduction and internal fixation through an extraoral approach. Two weeks after surgery, follow-up examination shows cellulitis and purulent drainage from the incisions. Which of the following factors most likely put this patient at an increased risk for this outcome? A) Antibiotic selection B) Choice of surgical approach C) Number of mandibular fractures D) Patient age E) Time between injury and definitive treatment
The correct response is Option C. Management of complex mandibular fractures carries a high risk for hardware complications and infection. Management decisions may be nuanced and patient-specific, but large retrospective case series clearly delineated patient and treatment factors that increased the risk for infectious complications. Increasing number of fractures occurring within the injury complex correlates with an increased risk for complications and postoperative infection. In this patient with a three-segment fracture pattern, the risk for infectious complication is increased to as high as 20%. The majority of facial fractures undergoing open reduction and internal fixation are treated through an intraoral approach to avoid external scarring or risk to the facial nerve, but this approach is associated with an increased risk for infection compared with external, transcutaneous approaches, likely due to exposure of the fracture and hardware to saliva. The use of a combined intraoral/extraoral approach is associated with the highest infection risk, although it is difficult to know if the association is due to the use of these approaches or the complexity of injuries necessitating this treatment. Despite general consensus that fractures of the dentate mandible should be treated expeditiously, evidence has demonstrated that delayed treatment is not associated with an increased risk for infection, allowing for delayed management in complex polytrauma patients. Evidence indicates that treatment with antibiotics prior to operative intervention for a displaced fracture of the dentate mandible decreases the risk for infection. Use of clindamycin has been shown to be associated with a higher rate of infection, likely due to developing resistance. The selection of antibiotic in this case was appropriate. Finally, medical comorbidities and tobacco use are associated with increased risk for infectious complications in an adult cohort, while age of the patient is not.
63
A 22-year-old man presents to the emergency department 1 hour after he sustained a penetrating injury to the neck. The patient reports “tasting blood.” On physical examination, there is a 1-cm vertical laceration at the left mandibular angle, nonexpanding retromandibular hematoma, and stable airway and hemodynamics. Which of the following is the most appropriate emergent management? A) Admission to the hospital for serial evaluations B) Radiologic imaging and intervention C) Surgical exploration at bedside D) Surgical exploration in the operating room E) Triple endoscopy in the operating room
The correct response is Option B. The patient presents with a zone III stable injury. The best initial practice is radiologic imaging with angiography and possible covered stent treatment for an internal carotid artery injury, or embolization of an external carotid artery branch in the retro- or parapharyngeal space. Secondary management may include a triple endoscopy or swallowing study to evaluate for a pharyngeal injury that can be treated with observation versus operative closure. Immediate surgical exploration for a stable zone III injury is not recommended due to the difficulty with exposure. Zone I includes the area between the cricoid cartilage and the manubrium and clavicles. Surgical exploration in this area can be technically challenging. Zone II includes the area between the angle of the mandible and the cricoid cartilage. This is the largest zone and is associated with the highest rate of injury to the neck. Surgical exploration in this area is not challenging, and exposure is easier than in zone I or zone III. Zone III includes the area between the angle of the mandible and the base of the skull. This area is often difficult to expose surgically and examine clinically, and it is difficult to determine the extent and depth of injury. 2024
64
A 40-year-old woman is evaluated because of an enlarging right cheek mass. CT scan shows a well-circumscribed tumor arising from the superficial parotid gland. Which of the following tumors is most likely in this patient? A) Cystadenoma B) Mucoepidermoid carcinoma C) Myoepithelioma D) Pleomorphic adenoma E) Warthin tumor
The correct response is Option D. Salivary gland tumors are characterized by diverse histologic appearances and variable biologic behavior. These tumors can be benign or malignant, and malignancies can be either primary or metastatic. It may also be difficult to distinguish between the different tumor types, especially on fine-needle aspiration. The rule of 80s regarding salivary gland tumors has been consistently backed up by the literature time and time again and describes the following: 80% of all salivary gland tumors are in the parotid 80% of parotid tumors are benign 80% of the benign parotid tumors are pleomorphic adenomas Warthin tumor, myoepithelioma, and cystadenoma are all benign parotid gland tumors but occur at a lower frequency than pleomorphic adenomas. Mucoepidermoid carcinoma is the most common primary malignant tumor of the parotid and may appear similar to pleomorphic adenoma on CT scan. 2024
65
A 32-year-old man presents with bilateral tissue masses proximal to the angle of the mandible. History is otherwise unremarkable. The patient reports no numbness, tingling, or any other symptoms. On physical examination, the affected area is more prominent when the patient clenches his jaw, and the incisal opening is normal. Which of the following studies would confirm this patient’s diagnosis? A) Biopsy of the mass B) Cephalometric analysis C) MRI D) X-ray study of the mandible
The correct response is Option C. The patient has masseter hypertrophy and the gold standard study is CT/MRI. Plain x-rays would not be descriptive enough to confirm the diagnosis. Biopsy is invasive and not necessary to aid in clinical suspicion. Diagnosis is often made based on clinical examination, and further imaging may not be necessary. This is an issue with the soft tissue, and cephalometric analysis would not be appropriate for masseter hypertrophy. 2024
66
A 65-year-old man undergoes right-sided total maxillectomy with resection of the entire orbital floor for T4 maxillary sinus cancer. The orbital floor is reconstructed with titanium mesh, and an anterolateral thigh free flap is used to reconstruct the maxilla and palate. Immediately postoperatively, the patient reports decreased vision on the right side; he can only see light and dark through the right eye. Which of the following is the most likely diagnosis? A) Enophthalmos B) Entrapment of the extraocular muscles C) Hardware exposure D) Orbital dystopia E) Traumatic optic neuropathy
The correct response is Option E. Traumatic optic neuropathy may result from direct injury to the optic (II) nerve or from traction on the globe that stretches the optic nerve during resection or reconstruction. Imaging is warranted to rule out impingement of the optic nerve by graft or hardware, which should be removed. High-dose corticosteroids are usually recommended. Prognosis for improvement is often poor. Entrapment of extraocular muscles from reconstruction can occur, but it is usually associated with diplopia secondary to restricted movement of the globe and sometimes pain from impingement of graft or hardware on the intraorbital soft tissues. Enophthalmos is not uncommon after extensive orbital wall resection but would not be associated with decreased vision. Similarly, orbital dystopia (eyes at different vertical levels) is not uncommon after extensive orbital wall resection. Usually, the globe is lower (hypophthalmos) on the affected side and may cause diplopia. Hardware exposure is usually not seen in the immediate setting after surgery and is unlikely to be associated with decreased vision in the acute setting. 2024
67
A 68-year-old woman with laryngeal cancer undergoes laryngopharyngectomy with bilateral neck dissection. The hypopharynx is reconstructed with a tubed anterolateral thigh free flap anastomosed to the left transverse cervical artery and vein. On postoperative day 2, nasogastric tube feedings are initiated, and shortly after, physical examination shows swelling of the left neck with no overlying erythema. Laboratory studies show a leukocyte count within the reference range and stable hematocrit. Over the next 24 hours, approximately 150 mL of milky white fluid is collected from the neck using a closed suction drain. Administration of which of the following is most appropriate at this time? A) Amoxicillin with clavulanate B) Botulinum toxin type A C) Chewing gum with sorbitol D) Glycopyrrolate E) Octreotide
The correct response is Option E. Chyle leak is a rare but difficult to manage complication following neck dissection. The leak may have also arisen from dissection of the transverse cervical artery and vein used in this case as recipient blood vessels for the anterolateral thigh free flap. The thoracic duct travels through the neck and terminates in the left internal jugular vein or the left subclavian vein in the supraclavicular region. There is also a right accessory lymphatic duct that can be injured during right neck dissection. Postoperative chyle leaks are usually identified after tube feeding is commenced. Laboratory analysis will show the presence of chylomicrons and a triglyceride concentration greater than 110 mg/dL. Multiple approaches to treatment have been recommended, including holding of feeds, use of total parenteral nutrition, or use of low-fat, medium-chain triglyceride (MCT) oral or tube feeds, since MCTs are not absorbed from the gut into the lymphatic system but instead are absorbed directly into the portal circulation. Octreotide is a long-acting somatostatin analogue and has also been used to reduce the formation of chyle. The proposed mechanism of action in the treatment of chyle leaks is decreasing absorption of triglycerides and inhibiting splanchnic circulation and gastrointestinal motility, all factors that affect lymph flow. Octreotide is given as a subcutaneous injection. Large leaks or those recalcitrant to conservative therapy are usually re-explored where the duct is identified and either repaired or ligated. In some cases, a flap such as the pectoralis major muscle pedicled flap is placed over the area of thoracic duct injury to help seal off any residual leakage. Sialogogues (such as chewing gum with sorbitol or xylitol), increased hydration, warm compresses, and massage are initial treatments used when the salivary gland duct is blocked. Glycopyrrolate can be used to help decrease salivary secretion when there is injury to the salivary gland causing a sialocele, such as after parotidectomy. Botulinum toxin type A injection is used to treat Frey syndrome and may be tried in salivary leaks as well. Antibiotics, such as amoxicillin with clavulanate, are usually given when a wound infection or fistula is a suspected cause of swelling. In this case, there is no erythema and drainage is consistent with chyle leak rather than an abscess or fistula. 2024
68
A 37-year-old man with a scalp sarcoma is scheduled to undergo calvarial bone resection. Compared with using other cranioplasty materials, using polyetheretherketone for immediate calvarial reconstruction is most likely to result in which of the following disadvantages? A) Difficulty adjusting implant size B) Exothermic curing process C) Increased likelihood of fracture D) Prone to resorption E) Radiographic artifacts
The correct response is Option A. Polyetheretherketone (PEEK) is a polymer material that must be milled or three-dimensionally printed to create implantable alloplastic replacements for human bone. Because of this, PEEK cranioplasty implants are usually custom-made based on CT scan data to fix pre-existing defects. When used for immediate reconstruction, the implant may not precisely fit the defect, especially if the dimensions of the defect changed based on intraoperative findings. While PEEK implants that are too big may be reduced with a cutting bur, implants that are too small cannot be adjusted to fit the calvarial defect. Advantages of PEEK include its radiolucency, strength, light weight, and stiffness similar to bone, which minimizes stress shielding. Titanium mesh causes significant artifacts on radiographic images, which may interfere with tumor surveillance. Calcium hydroxyapatite cements have a tendency to be brittle and fracture. Methylmethacrylate is applied in a paste form and the curing process is exothermic, requiring continuous irrigation to prevent burning of the surrounding tissues. Autologous bone grafting is felt by many to be the ideal material for cranioplasty due to low complication rates, but it is obviously available only in limited quantities and resorption may occur. 2024
69
A 60-year-old man with a history of smoking is evaluated because of a 2-year history of a lesion in his mouth. He smokes two packs of cigarettes daily. Physical examination shows white plaques with speckled ulcerations along the floor of the mouth. Examination of a specimen obtained on biopsy discloses leukoplakia with histologic evidence of dysplasia. Which of the following associated malignancies is most likely to be seen in this patient? A) Basal cell carcinoma B) Cutaneous T-cell lymphoma C) Malignant melanoma D) Merkel cell carcinoma E) Squamous cell carcinoma
E The most common malignancy associated with leukoplakia is squamous cell carcinoma. About 10% of leukoplakic lesions progress to malignancy, and frequency is related to the site of occurrence. Leukoplakia of the floor of the mouth is associated with over 60% squamous cell carcinoma in situ or invasive squamous cell carcinoma, whereas leukoplakia of the buccal mucosa is almost always found to be benign. Speckled gray or white patches are more likely to develop into carcinoma. Predisposing factors include smoking, alcohol, and oral snuff. Oral leukoplakia is classified as a premalignant lesion because it is caused by exposure to the same agents that induce squamous cell carcinoma, namely smoking, alcohol, irritation from dentures or a jagged tooth, and occurrence adjacent to squamous cell carcinoma. Leukoplakia with dysplasia is not associated with other melanoma or nonmelanoma malignancies of the mouth, such as basal cell carcinoma, melanoma, Merkel cell carcinoma, or cutaneous lymphoma. 2024
70
A 45-year-old woman undergoes Mohs micrographic surgery for a lower eyelid basal cell carcinoma. This results in a 40% full-thickness defect of the middle aspect of the lower eyelid margin and tarsal plate. Margins are free of disease. Which of the following is the most appropriate reconstruction? A) Coverage of the defect with a laterally based Tripier flap B) Primary closure with canthoplasty C) Reconstruction with a composite graft D) Reconstruction with a Tenzel semicircular flap, lateral cantholysis, and canthoplasty E) Reconstruction with a two-stage Cutler-Beard flap and auricular cartilage graft
The correct response is Option D. For smaller full-thickness defects of the upper and lower eyelid, pentagonal excision and primary closure removes the resultant dog ear and reconstructs the defect with minimal secondary deformity. This is ideal for defects involving 20 to 30% of the length of the eyelid. In the described case, a 40% defect in a 45-year-old woman will result in excess tension and potential bowstringing of the eyelid over the convexity of the globe, which would result in lid malposition. Relief of tension with a lateral cantholysis and recruitment of lateral tissue using a Tenzel semicircular flap is necessary to achieve primary closure. This technique is ideal for defects between 40 to 60% of the eyelid width. The Tripier flap is a laterally based myocutaneous flap used to recruit tissue from the ipsilateral upper eyelid for anterior lamellar reconstruction of the lower eyelid. It would not address the posterior lamellar or tarsal plate deficiency. When combined with a full-thickness skin graft for anterior lamellar reconstruction, it addresses full-thickness defects of the lower eyelid but is excessive given the size of the described defect. Finally, the Cutler-Beard flap is a full-thickness flap of the lower eyelid that can be augmented with a cartilage graft to reconstruct near-total defects of the upper eyelid in a two-stage approach. This procedure is used for upper eyelid reconstruction and is not appropriate for this defect. A 40% defect is too large for primary closure or a composite graft. 2024
71
A 55-year-old man with a history of polymyalgia rheumatica presents for follow-up evaluation one week after he received a diagnosis of Bell palsy. Treatment with prednisone 60 mg daily has not improved his symptoms. Physical examination shows excessive tear production, bilateral ptosis, and persistent facial weakness. The patient reports increasingly severe pain in the mastoid area and an altered sense of taste. Which of the following findings most likely requires further investigation? A) Altered sense of taste B) Bilateral ptosis C) Excessive tear production D) Persistent facial weakness E) Worsening pain in the mastoid area
The correct response is Option B. Bell palsy is typically associated with postauricular pain, sudden unilateral facial weakness, and excessive tear production. Approximately half of patients present with postauricular pain in the mastoid area. Patients may also experience a decreased or altered sense of taste, or unilateral ptosis. Bilateral ptosis is not caused by Bell palsy and should be investigated. 2024
72
A 40-year-old woman presents to the emergency department 2 hours after she was stabbed in the left temporal area. On physical examination, the patient is able to raise the eyebrows normally, open and close the eyes, and depress the lips. There is significantly decreased function when trying to smile, puff the cheeks, and purse the lips. Which of the following is the most appropriate next step in management? A) Administration of oral corticosteroids B) Electromyography C) Facial nerve exploration D) MRI E) Primary wound closure
The correct response is Option C. This patient has damage to the buccal branches of the facial (VII) nerve. The literature supports immediate exploration and repair of the lacerated nerve branches within 72 hours, primarily or with grafting. The negative impact of facial paresis or paralysis is significant for adults and children, from the functional and emotional perspective, with increased risk for depression for both groups. 2024
73
A 28-year-old man is evaluated because of a 9-month history of right-sided facial paralysis due to Bell palsy. Dynamic reinnervation using a motor nerve to masseter transfer is planned. The branch of the nerve to the masseter muscle targeted by this procedure is located between which of the following structures? A) Middle and deep lobes of the masseter muscle B) Parotideomasseteric fascia C) Parotid gland and masseter muscle D) Superficial and middle lobes of the masseter muscle E) Superficial musculoaponeurotic system and masseter muscle
The correct response is Option A. The nerve to the masseter muscle descends obliquely beneath the zygomatic arch, coursing toward the oral commissure. The nerve ramifies as it approaches the deep surface of the masseter and through the substance of the muscle. The target branch is found between the deep and middle lobes of the masseter. The masseter is composed of superficial, middle, and deep lobes, which originate on the zygomatic arch and insert on the ramus and angle of the mandible. The mandibular nerve exits the skull base through the foramen ovale to enter the infratemporal fossa, the space deep to the zygomatic arch. The motor branch to the masseter muscle then arises within the infratemporal fossa and passes laterally toward the zygomatic arch. At the zygomatic arch, it descends in an oblique course through the sigmoid notch of the mandible. It then proceeds anteriorly and inferiorly toward the ipsilateral oral commissure. Before entering the undersurface of the muscle, the nerve consists of two branches in 47% of specimens, one branch in 25%, and three branches in 25% of cases. Often, only one of the branches is of sufficient caliber for grafting, averaging 2 mm in caliber. However, the nerve is a high-quality donor source, containing between 1500 and 2000 motor neurons. 2024
74
A 70-year-old woman is evaluated because of total right-sided facial paralysis. Five years ago, she underwent resection of a right vestibular schwannoma (acoustic neuroma) with preservation of the facial nerve. Preoperative examination showed decreased facial movement. There has been no facial movement or tone since operative intervention. The patient desires dynamic restoration of her smile. Which of the following procedures is most appropriate for this patient? A) Cable nerve grafting B) Cross facial nerve grafting C) Fascial suspension with tensor fascia lata graft D) Lengthening temporalis myoplasty E) Masseteric nerve to facial nerve transfer
The correct response is Option D. Lengthening temporalis myoplasty, or Labb procedure, is a surgical technique for dynamic restoration of the smile. The Labb procedure involves division of the temporalis muscle insertion on the coronoid process of the mandible, partial detachment of the muscle from the cranium, and attaching it to the perioral muscles. In the original description, the zygomatic arch was osteotomized for ease of dissection and then restored with rigid fixation. In later modifications, the zygomatic arch is preserved. Results are generally pleasing, and this technique is used by surgeons as an alternative to functional free muscle transfer, which requires microvascular anastomoses and reinnervation of the donor muscle from a recipient nerve. Fascial sling suspension with a tensor fascia lata graft is not a dynamic procedure. The goal of this procedure is to elevate the oral commissure and nasolabial area to prevent ptosis and oral incompetence. Cable nerve grafting, cross facial nerve grafting, and masseteric nerve to facial nerve transfer often give superior results to muscle transfers and static reanimation. However, given the long time period that has passed since denervation, motor endplate degeneration on the facial musculature has occurred and reanimation of the native facial musculature is not possible. Additionally, cable nerve grafting is not possible given the intracranial location of the lesion. 2024